Sesap Oncology

You might also like

Download as docx, pdf, or txt
Download as docx, pdf, or txt
You are on page 1of 81

SESAP- oncology

A 50-year-old man has a 1-cm left thyroid nodule palpated on physical


exam. There is no family history of endocrine disorders or malignancy.
Laboratory values show normal thyroid-stimulating hormone. Thyroid
ultrasound shows a solitary 1.3-cm hypoechoic, solid, left thyroid nodule
with microcalcifications. Fine needle aspiration biopsy under ultrasound
guidance shows neuroendocrine cells with background material staining
positive for amyloid consistent with medullary thyroid cancer.
The next most appropriate step in his management is
A
measurement of serum calcitonin.
B
measurement of serum thyroglobulin.
C
fluorodeoxyglucose PET scan.
D
somatostatin receptor scintigraphy.
E
menin mutation analysis.

A
measurement of serum calcitonin.

Medullary thyroid cancer is an uncommon thyroid malignancy arising from


the calcitonin-secreting parafollicular cells.

Most cases are sporadic, but 20–25% are familial and associated with a RET
mutation.

A fine needle aspiration diagnosis of medullary thyroid cancer requires the


following preoperative work-up:
- comprehensive neck ultrasound to evaluate for lymphadenopathy,
- measurement of serum calcitonin and
- carcinoembryonic antigen,
- RET mutation analysis, and
- exclusion of pheochromocytoma by either
o (1) negative RET mutation analysis and negative family history
or
o (2) negative plasma free or 24-hour metanephrines and
normetanephrines.

CT imaging of the neck, chest, and abdomen is obtained if


- cervical lymphadenopathy is present or
- the calcitonin is greater than 400 pg/mL. 

Serum thyroglobulin is not a part of the evaluation of a patient with


medullary thyroid cancer; it is used as a postoperative tumor marker in
patients with thyroid cancer of follicular cell origin.

Fluorodeoxyglucose PET and somatostatin receptor scintigraphy are not


recommended for the routine initial evaluation of a patient with medullary
thyroid cancer.

The menin gene is mutated in patients with multiple endocrine neoplasia


type 1 (MEN-1).
Medullary thyroid cancer is not associated with this genetic abnormality.

==============================================================================

A 50-year-old man has a 1-cm left thyroid nodule palpated on physical


exam. There is no family history of endocrine disorders or malignancy.
Laboratory values show normal thyroid-stimulating hormone. Thyroid
ultrasound shows a solitary 1.3-cm hypoechoic, solid, left thyroid nodule
with microcalcifications. Fine needle aspiration biopsy under ultrasound
guidance shows neuroendocrine cells with background material staining
positive for amyloid consistent with medullary thyroid cancer.
He has a negative preoperative metastatic workup and negative genetic
testing. The next most appropriate step in his management is
A
left hemithyroidectomy.
B
total thyroidectomy.
C
total thyroidectomy with prophylactic central neck dissection.
D
total thyroidectomy and removal of any enlarged lymph nodes in the neck.
E
total thyroidectomy and modified left neck dissection.

C
total thyroidectomy with prophylactic central neck dissection.

Hemithyroidectomy is not an appropriate treatment for patients with


medullary thyroid cancer, because, even in sporadic cases, the incidence of
multifocal disease can be as high as 22%.

Medullary thyroid cancer has a high rate of lymph node metastases in the
central compartment that are not always detected on preoperative neck
ultrasound or intraoperatively by the surgeon.

Thus, the recommended surgical treatment for a patient with medullary


thyroid cancer is total thyroidectomy with prophylactic central neck
dissection.

This involves clearance of the entire central compartment of the neck.

“Berry picking” of enlarged lymph nodes is not recommended. 

The utility of prophylactic lateral compartment neck dissection is debated.

The most recent guidelines for the treatment of medullary thyroid cancer
published by the American Thyroid Association do not advocate prophylactic
lateral compartment neck dissection, because with the advent of high
resolution neck imaging, the lateral compartment of the neck can be imaged
preoperatively with high sensitivity.

If an abnormal lymph node is seen on ultrasound, it should undergo fine


needle aspiration biopsy, and, if positive, a lateral neck dissection should be
done.

Thyroid-stimulating hormone (TSH) suppression is part of the treatment of


patients with thyroid cancers of follicular cell origin.

Patients with medullary thyroid cancer should receive thyroid hormone at a


dose that results in a TSH within the normal range postoperatively.
==============================================================================

A 60-year old man presents with a pruritic lesion on his arm that he was
initially told was an allergic reaction (figure 1). He is otherwise
asymptomatic. Examination discloses no other skin lesions and no
lymphadenopathy. A biopsy shows T-cell lymphoma. The next best step is

[+] CLICK IMAGE TO ENLARGE


FIGURE 1

A
observation.
B
radiation.
C
sentinel node biopsy.
D
multiagent chemotherapy.
E
PET/CT scan.

E
PET/CT scan.

Seventy-five percent of cutaneous lymphomas are


- non-Hodgkin, T-cell lymphomas,
and two-thirds of those are either mycosis fungoides
(also called granuloma fungoides or Alibert-Bazin syndrome) or Sézary
syndrome.

These lesions are age related and are increasing in incidence.


They are more common in men than women and African Americans than
whites.

They present as patches or plaques that are usually pruritic and may be
misdiagnosed as allergic lesions or psoriasis.

Cutaneous lymphomas can persist for months or years before a diagnosis is


made.

Biopsy shows malignant T-cell clones that are typically


- CD3+, CD4+, CD8–, and
- lack T-cell markers such as CD2, 5, and 7.

The lesions are staged by the tumor, nodes, metastases, blood classification
(TNMB) system, as shown in table 1.

Sézary cells are circulating tumor cells with typical grooved, lobulated nuclei.

Along with the presence of erythroderma, they differentiate Sézary


syndrome from the more indolent mycosis fungoides.

In the absence of abnormally enlarged nodes, lymph node biopsy is not


indicated.

The patient presented has disease covering less than 10% of the body
surface and no lymphadenopathy.

He may therefore have limited stage mycosis fungoides, which is the most
common stage at presentation and generally carries an excellent prognosis,
but requires staging with PET/CT and peripheral blood smear to rule out
visceral disease and leukemic cells in the blood.

Treatment for limited disease should focus on skin-directed therapies such


as
- topical steroids,
- radiation, and
- phototherapy;
observation is also appropriate in some selected early stage patients.

Because it does not improve disease-free or overall survival in early-stage


patients and can be associated with significant toxicity, multiagent
chemotherapy with or without radiation should be reserved for more
advanced stages.
Table 1. Tumor, nodes, metastases, blood classification for cutaneous lymphoma

==============================================================================

A 40-year-old woman presents with a swollen anterior thigh. Physical exam


reveals a palpable, nontender mass without overlying skin changes. The MRI
is shown in figure 1. Core biopsies are read as atypical lipomatous tumor.
The best next step is

[+] CLICK IMAGE TO ENLARGE


FIGURE 1

A
narrow-margin resection.
B
radiation.
C
chemotherapy.
D
compartment resection with wide margins.
E
open incisional biopsy.

A
narrow-margin resection.
Liposarcomas (LPSs) are most commonly found in the thigh and
retroperitoneum.

They can remain asymptomatic until quite large.

LPSs have a diverse array of subclassifications; in recent years it was been


recognized that 40–45% of LPSs are well differentiated (WDLPSs) and do not
show metastatic potential unless there is a dedifferentiated component
found on pathology.

WDLPSs can be distinguished genetically by amplification of the proto-


oncogenes MDM2, CDK4, and HGMA2, all located on chromosome 12q. 

Because excision is usually curative for WDLPSs, the term atypical lipomatous


tumor (ALT) was introduced to describe WDLPSs that are
- very well differentiated and
- often located superficially or on the extremities.

Histologically, ALTs are characterized by mature fat with a variable number


of atypical, often enlarged and pleomorphic nuclei but with low cellularity
and rare mitoses.

Core biopsy for these lesions is preferred; open biopsy should be done only if
core biopsy is unavailable or nondiagnostic.

Limb salvage should be the goal of resection; in fact, WDLPSs/ALTs may be


the one sarcoma subtype for which narrow margin excision or even
debulking is appropriate when wide negative margins cannot be obtained.

In the patient shown, the lesion abuts the femoral neurovascular bundle;
therefore, narrow-margin resection is the best option. 

Most series show a risk of dedifferentiation for recurrent WDLPSs/ALTs of


less than 5%.

Adjuvant radiation may be of benefit after R1 resection or for recurrent


disease.

Chemotherapy is reserved for any LPSs with unresectable systemic disease;


distant spread is not seen in most series of ALTs.
In fact, for the general surgeon, the real quandary is differentiating between
ALTs and benign lipomas preoperatively.

Clues that a lipomatous tumor may be an ALT and not just a benign lipoma
include patient age
- older than 55,
- size greater than 10 cm,
- recurrence, and
- extremity location.

==============================================================================
An 80-year-old woman had a mastectomy and radiation for breast cancer 20
years ago. She has chronic lymphedema in the arm, which she treats with
compression sleeve. She presents with a 2-month history of a progressive,
irregular, purple-colored lesion of the arm (figure 1). The most likely
diagnosis is

[+] CLICK IMAGE TO ENLARGE


FIGURE 1

A
recurrent breast cancer.
B
follicular lymphoma.
C
compression sleeve trauma.
D
angiosarcoma.
E
melanoma.

D
angiosarcoma

Chronic lymphedema has a permissive effect on the development of certain


malignancies, thought to be due to the creation of a field of
immunosuppressed tissues rich in growth factors and immunosuppressive
cytokines (such as IL-10) that prevent the maturation of antigen-presenting
dendritic cells.

The most common example of cancer developing in lymphedematous tissues


is the scenario of angiosarcoma developing in a lymphedematous arm after
prior treatment for breast cancer; the adjusted odds-ratio of lymphedema
causing angiosarcoma is 59.

This phenomenon is known as the Stewart-Treves syndrome.

The photograph demonstrates the reddish-blue macules and nodules that


eventually coalesce typical of these tumors (figure 2).

Although the incidence of such severe lymphedema after breast cancer


treatment is decreasing, it is being replaced by
massive-localized lymphedema (MLL) secondary to obesity.

MLL is linked to an increased risk of angiosarcoma.

Other tumors linked to chronic lymphedema after breast cancer therapy are
rarer; these include
- Kaposi sarcoma,
- nonmelanoma skin cancer,
- melanoma, and
- follicular lymphoma.

Breast cancer recurrence is not linked to lymphedema.

The most common sites for metastatic breast cancer include the
- lung/pleura,
- bones,
- liver,
- brain, and
- nonaxillary nodes.

Although cutaneous metastases from breast cancer are common, most occur
in the skin associated with the primary tumor; cutaneous metastases from
breast cancer outside the breast are rare.

Complications related to compression sleeves usually consist of rashes, pain,


and (rarely) cellulitis.

Figure 2. Stewart-Treves syndrome. The photograph shows the reddish-blue macules and nodules that
eventually coalesce, typical of these tumors.

==============================================================================

A 64-year-old healthy woman presented with a rectal mass located


approximately 6 cm from the anal verge and shown in figure 1. A biopsy
confirmed a poorly differentiated adenocarcinoma, and staging workup
shows an uT3N0M0 lesion. Which of the following is the most appropriate
treatment option?

[+] CLICK IMAGE TO ENLARGE


FIGURE 1

A
Low anterior resection followed by chemotherapy
B
Transanal excision
C
Neoadjuvant chemoradiation followed by low anterior resection
D
Abdominal perineal resection
E
Low anterior resection followed by radiation therapy

C
Neoadjuvant chemoradiation followed by low anterior resection

For patients with upper-third rectal cancer (8–12 cm from the anal verge), it
is acceptable to approach the cancer like a colon cancer—mainly to offer up-
front surgery followed by adjuvant therapy, as deemed appropriate.

For patients with mid to lower early rectal cancer (T1–2, N0, M0, or stage I
disease), radical surgery alone is sufficient because the 5-year survival rate
can exceed 90%.

However, for patients with clinical stage II (cT3–4, N0, M0) or III (any T,
N+, M0) disease that is located in the mid to lower rectum, management has
evolved from surgery only into a multimodality therapy using chemotherapy,
radiotherapy, and surgery.

Unlike colon cancer, neoadjuvant chemoradiation therapy for rectal cancer is


recommended for stage II disease. 

The 2004 German trial redefined the sequence of therapy for patients with
stage II/III rectal cancer.

More than 800 patients with clinical stage T3/T4 or node-positive disease
were randomized to receive either neoadjuvant chemoradiation therapy
(neoCRT) or adjuvant chemoradiation therapy (postCRT).

Total mesorectal excision, a technique that removes the LN-bearing


mesorectum using sharp dissection, was performed in all patients.

At a median follow-up of 46 months, although


- overall survival was not statistically significant between the 2 groups
(76% in neoCRT group vs 74% in the postCRT group; p = .80), the 5-
year cumulative incidence of
- local relapse was 6% in the neoCRT group versus 13% in the postCRT
group (p = .006).
There was no difference in the postoperative complication rate between the
2 groups, which alleviated the concern of increased surgical complications in
a radiated field.

In addition, the preoperative group had


- lower incidence of grade 3 or 4 acute toxicities (27% vs
40%; p = .001),
- lower long-term toxic effects (14% vs 24%; p = .01), and
- higher sphincter preservation rate in patients for whom an
abdominoperineal resection would have been required as deemed by
the surgeon (39% vs 19%; p = .004).

A 2012 update of this study with a median follow-up of 11 years confirmed


the durability of these results. 

Transanal excision is limited to a select group of T1N0M0 lesions and is not


appropriate for this patient.

Abdominal perineal resection may be too radical of an approach for first-line


therapy for a lesion at 6 cm where sphincter preservation is possible.

Figure 1. Rectal mass.

==============================================================================

Which of the following statements about Merkel cell carcinoma of the skin is
true?
A
It is less aggressive than melanoma.
B
Treatment is excision to 1-cm margins.
C
It does not spread via regional lymph nodes.
D
It is a neuroendocrine tumor.
E
There is no role for chemoradiation.

D
It is a neuroendocrine tumor.

Merkel cell carcinoma is an aggressive neuroendocrine tumor of the skin


characterized by rapid progression and intravascular and lymphatic spread.

It is more aggressive and has a lower survival rate than melanoma.

These lesions are associated with a synchronous or metasynchronous


squamous cell carcinoma 25% of the time.

Treatment is
- wide excision with
- 3- to 5-cm margins,
- sentinel node biopsy, and
- adjuvant radiation therapy.
Chemotherapy for regional disease (positive regional nodes) is also
recommended.

==============================================================================

Which of the following molecular tests should be performed on the tumor


specimen after surgical resection of any newly diagnosed stage II colon
cancer?
A
VEGF levels
B
p53 mutation
C
Microsatellite instability
D
BRCA1 mutation
E
EGFR mutation

C
Microsatellite instability
Until recently, all colorectal cancers were considered to have the same
cause, clinical characteristics, and treatment outcomes, but colorectal cancer
is being recognized as a complex and heterogeneous disorder. A number of
distinct molecular pathways to colorectal cancer are recognized.
One is the hereditary microsatellite instability (MSI) pathway found in
hereditary nonpolyposis colon cancer (HNPCC) or Lynch syndrome. HNPCC
patients and family members were initially identified by clinical features, but
routine molecular screening for patients with colorectal cancer for Lynch
syndrome has better sensitivity for detecting mutation carriers. Studies
demonstrate that testing for Lynch syndrome in all newly diagnosed patients
with colorectal cancer is cost-effective. An expert panel from the National
Comprehensive Cancer Network recommended that all patients with
colorectal cancer be screened for Lynch syndrome.
HNPCC tumors have 2 distinguishing characteristics: (1) microsatellite
instability, which is the expansion or reduction in the length of repetitive
DNA sequences (known as microsatellites) in the tumor DNA compared with
normal DNA, and (2) loss of 1 or 2 of the mismatch repair proteins in the
tumor compared with normal tissue. MSI is used as a surrogate marker of
HNPCC. Tumors with no instability are considered to be microsatellite stable
(MSS). A tumor is considered to have low MSI (MSI-L) when 1 reference
marker is mutated and High MSI (MSI-H) if 2 or more markers are altered. If
screening identifies a tumor as MSI-H, immunohistochemical staining is used
to detect the presence or absence of the protein products of the mismatch
repair genes. A missing protein suggests a mutation in the gene that codes
for that protein.
Molecular testing can identify possible targets for therapy. The epidermal
growth factor receptor molecule is critical for many known survival pathways
in colorectal cancer, and some tumors have increased copy numbers of
the EGFR gene. Although the amount of EGFR can be measured in the
tumor, these measurements do not predict clinical response to therapy that
blocks EGFR (e.g., cetuximab). Vascular endothelial growth factor (VEGF)
can also be measured in tumors and do correlate with outcome, however,
tumor levels have again not correlated with response to anti-VEGF therapy
(e.g., bevacizumab). Inherited defects in the tumor suppressor genes, p53,
and BRCA1, are responsible for Li-Fraumeni and breast-ovarian cancer
syndromes.
=====================================================================

A 60-year-old man presents with a 1.9-mm thick melanoma of his upper


back diagnosed by punch biopsy. Physical exam reveals no clinical
lymphadenopathy. The next best step in his management is
A
whole-body PET scan.
B
tumor testing for BRAF mutation.
C
Mohs surgery.
D
wide excision of primary tumor with 2-cm margins.
E
wide excision of primary tumor with 2-cm margins and sentinel lymph node
biopsy.

E
wide excision of primary tumor with 2-cm margins and sentinel lymph node
biopsy.

Clinically node-negative melanoma is treated with


- wide local excision of the primary tumor and
- sentinel lymph node biopsy (SLNB) in select patients.

Radial margins of the wide local excision are determined by Breslow depth.
For a Breslow depth
- less than or equal to 1 mm---1 cm radial margins

- greater than 1 mm but less than 2 mmradial margins of 1.5–2.0 cm

- greater than 2 mm-2-cm radial margins


SLNB is used to detect microscopic nodal metastases in high-risk patients.

SLNB is commonly recommended for melanomas with a Breslow depth


greater than or equal to 0.75 mm.

For tumors thinner than 0.75 mm, the indications for SLNB are less clear but
may include ulceration, mitotic figures, or other high-risk pathologic
features. 

Imaging of asymptomatic patients with early stage melanomas is not


recommended.

Only 50% of melanomas harbor a mutation in the BRAF gene, which results
in an altered BRAF protein.

The altered BRAF protein promotes the growth of melanoma through


changes in the MAP kinase/ERK pathway.

BRAF inhibitors (vemurafenib and dabrafenib) can slow the growth of


melanoma.
These inhibitors are currently approved for use in unresectable and
metastatic melanoma only. 

Mohs surgery is not recommended for the surgical treatment of invasive


melanoma.

==============================================================================

A 48-year-old woman recently diagnosed with small cell lung cancer of the
right lower lobe, metastatic to the liver, is admitted to the hospital for poorly
controlled diabetes, severe hypertension, and hypokalemia. Serum
adrenocorticotropic hormone (ACTH) level is 823 pg/mL (9–52 pg/mL).
Sinus petrosal sampling shows no abnormal elevation in ACTH level. Which
of the following is the best treatment plan?
A
Resection of primary tumor via right lower lobectomy
B
Chemotherapy and whole-brain radiation
C
Laparoscopic bilateral adrenalectomy
D
Medical management with octreotide, mifepristone, and ketoconazole
E
Stereotactic body radiotherapy to both adrenals

C
Laparoscopic bilateral adrenalectomy

This patient has ectopic Cushing syndrome, which is a well-described


paraneoplastic syndrome associated with small cell lung cancer.

These patients suffer from hypercortisolism due to adrenocorticotropic


hormone (ACTH) secretion by both the primary tumor and the metastatic
lesions.

Their symptoms include


- diabetes,
- hypertension (due to excess mineralocorticoid),
- immunosuppression, and
- metabolic disturbances such as hypokalemia.

All of these patients should have an ACTH-producing pituitary tumor ruled


out by sinus petrosal sampling.

When ectopic Cushing syndrome is related to metastatic cancer, and the


patient is otherwise believed to have a reasonable survival, the optimal way
to palliate their symptoms is with laparoscopic bilateral adrenalectomy.

Laparoscopic bilateral adrenalectomy is a safe and effective treatment for


ectopic Cushing, relieving most of these patients of their often incapacitating
symptoms.

Because this particular patient has liver metastases, resection of the primary
tumor via right lower lobectomy would be unlikely to resolve her ectopic
Cushing.

Chemotherapy and whole brain radiation is often the treatment of choice for
metastatic small cell lung cancer, with some patients able to achieve
prolonged survival.

However, it is unlikely that this patient would tolerate chemotherapy with


her acute hypercortisolism.
Consequently, laparoscopic bilateral adrenalectomy should be the logical first
step in treatment.

Medical treatment of ectopic Cushing syndrome includes the use


- octreotide (a somatostatin analogue),
- mifepristone (a cortisol receptor antagonist), and
- ketoconazole (inhibitor of cortisol synthesis).

Although each of these medications is useful to a degree, none is as effective


as laparoscopic bilateral adrenalectomy.

Although some studies have shown stereotactic body radiation therapy


(SBRT) to be effective at treating limited metastatic tumor to the adrenals,
no studies have shown SBRT to be useful in blocking adrenal function in
ectopic Cushing syndrome.

==============================================================================

A 70-year-old woman with no personal or family history of cancer recently


developed a T2N1M0, well-differentiated adenocarcinoma of the ascending
colon. Which gene is most likely to be mutated in this patient?
A
MLH1
B
MSH2
C
TP53
D
APC
E
kRAS

D
APC

The progression of most sporadic colon cancer from adenoma to carcinoma


is characterized by the development of
- genomic instability and
- compromised DNA integrity.

In patients with sporadic colon cancer, the inactivation of the tumor


suppression gene APC is most commonly seen.

The gene TP53 is considered a tumor suppressor gene that encodes the
protein p53.

MLH1 and MSH2 are typically associated with familial cancers; these are
associated with Lynch syndrome (previously known as hereditary
nonpolyposis colon cancer).

These genes are rarely seen in sporadic colon cancer.

kRAS can be seen in some of these patients, although it is not a primary


driving force.

The understanding of these genetic changes is being used in an attempt to


predict outcome and determine better subsequent treatment plans.

==============================================================================

A middle-aged man presents with the lesion shown in figure 1 that has been
present for many years. The most important aspect of the management of
this lesion is

[+] CLICK IMAGE TO ENLARGE


FIGURE 1

A
avoidance of ectropion.
B
preoperative imiquimod.
C
clear surgical margins by histopathology.
D
preoperative adjuvant radiation therapy.
E
use of a forehead advancement flap for closure.

C
clear surgical margins by histopathology.

The primary goal of all oncologic surgical therapy is complete removal of the
tumor.

This involves local resection to clear margins of an appropriate width.

Ectropion is problematic but can be minimized with the use of skin grafts.

A good color match of the skin used for closure is a consideration, but of
secondary importance.

Local tissue transfers (i.e., advancement flaps) are not recommended until
complete pathologic margin assessment of all peripheral and deep margins
has determined that there is no residual tumor.

Adjuvant radiation therapy results in a higher recurrence rate when used for
salvage of inadequate surgical excision.

Radiation therapy may be considered when patient preference or


considerations for cosmesis and function supersede the goal of complete
surgical removal of the tumor.

Radiation therapy is generally reserved for recurrence and not as standard


adjuvant therapy when extirpation of the primary tumor is complete.

Perineural involvement by tumor may be an exception.

Topical imiquimod is an immune-enhancing agent, but the exact mechanism


of action is not known.
It is approved for use in precancerous lesions, such as actinic
keratosis, genital warts, and some basal cell carcinomas, particularly
those that are not surgical candidates.

However, surgical excision remains the preferred approach to the


management of cutaneous malignancies.
==============================================================================

A 70-year-old man presents with a 2-month history of episodic diarrhea and


associated abdominal cramping with periods of “feeling flush.” He has no
previous surgical history. He is taken to the operating room for exploratory
laparotomy for a high-grade bowel obstruction. During the induction of
anesthesia, he develops significant hypertension, flushing, and tachycardia.
Which of the following statements about this condition is true?
A
The duodenum is the most common small intestinal location of this tumor.
B
Preoperative measurement of elevated urinary cortisol confirms metastatic
disease.
C
Metastatic disease beyond the small bowel is present in 10% of patients at
the time of diagnosis.
D
Multifocal small bowel involvement occurs in less than 5% of patients.
E
Combination therapy with everolimus plus octreotide improves progression-
free survival.

E
Combination therapy with everolimus plus octreotide improves progression-
free survival.

Carcinoid crises are rare life-threatening events involving cardiac instability.

Serotonin is thought to be one of the compounds capable of precipitating a


carcinoid crisis.

Serotonin is enzymatically inactivated in the liver into 5-hydroxyindoleacetic


acid (5-HIAA), which suggests that this patient likely has liver metastases
that secrete serotonin directly into the systemic rather than portal
circulation.
Elevated urine 5-HIAA is suggestive of carcinoid syndrome, whereas
elevated urine cortisol is associated with Cushing syndrome. 

Carcinoid tumors of the jejunum and ileum are the third most common
primary site (after lung and rectum), with an annual incidence of 0.67 per
100,000, whereas the incidence of duodenal neuroendocrine tumors is 0.19
per 100,000.

Nearly 50% of patients with neuroendocrine tumors present with metastatic


disease, and multifocal small bowel involvement occurs in 25% of patients. 

Intravenous octreotide is used to reverse a carcinoid crisis.

In the RADIANT-2 trial, everolimus, an oral mTor inhibitor, plus long acting
repeatable (LAR) octreotide improved progression-free survival in patients
with carcinoid syndrome compared with placebo plus LAR octreotide.

==============================================================================

A 54-year-old man is found to have a retroperitoneal liposarcoma. Which of


the following is a prognostic factor associated with increased survival?
A
Postoperative chemotherapy
B
Microscopically clear margins
C
Multifocal disease
D
Preoperative chemotherapy
E
Preoperative radiation therapy

B
Microscopically clear margins

Retroperitoneal sarcomas (RPS) are relatively rare, accounting for less than
1% of adult cancers.
Therefore, robust prospective clinical trial data are absent for this disease.

Retroperitoneal liposarcoma is primarily treated with grossly complete


surgical resection of the tumor with en bloc resection of involved organs,
when feasible.

Local recurrence is the leading cause of death in patients with RPS.

Traditional TNM (tumor size, node status, and metastases) staging for
sarcomas was derived from analyses of extremity soft tissue sarcomas and
is less useful in staging RPS.

For RPS, tumor grade, invasion of adjacent structures, and histologic


subtype are all prognostic factors.

Other prognostic factors for RPS include age of the patient, primary versus
recurrent, multifocality, and completeness of resection (R0/R1 vs R2).

Size was found to be prognostic in some analyses but not in others, likely
because most RPS are large when discovered.

Chemotherapy and radiation therapy to treat RPS remains controversial.

A few published studies address radiation in the treatment of RPS, and there
is great variation as to whether it is used at all or, if used, whether it should
be used pre-, intra-, or postoperatively.

Although prospective and retrospective studies evaluated radiation in the


treatment of RPS, none conclusively showed increased survival.

At least one trial was opened to evaluate this question and closed due to
lack of accrual.

The use of chemotherapy in the treatment of RPS does not improve survival
or downsize the tumor if given preoperatively.

Currently, there are no published randomized trials of preoperative or


postoperative chemotherapy for RPS.

==============================================================================

A 75-year-old woman with familial polyposis who has had a total colectomy
complains of the new onset of intermittent abdominal pain, which is worse
with eating. An MRI shows 2 masses in the mesentery (figure 1). Which of
the following is the most likely diagnosis?

[+] CLICK IMAGE TO ENLARGE


FIGURE 1

A
Desmoid
B
Liposarcoma
C
Carcinoid
D
Lymphoma
E
Leiomyoma

A
Desmoid

The relative risk of developing desmoid tumors is much higher in patients


with familial adenomatous polyposis (FAP) compared with the general
population.

Desmoids occur in 7.5–16% of FAP patients.

Gardner syndrome, considered a variant of FAP, includes


- extracolonic manifestations such as
o osteomas
o desmoid tumors.
Given her history of FAP and the location of the masses, a diagnosis of
desmoid is most likely.

Diagnosis of desmoids and other aggressive fibromatosis (AF) is made based


on clinical, radiological, and histological parameters.

AF/desmoids characteristically infiltrate deep tissue and muscles as opposed


to pushing the adjacent tissue, as is seen in most sarcomas.

MRI is an excellent tool for the identification and characterization of


mesenteric masses (figure 2, table 1).

On MRI, the lesions are infiltrative with an irregular or lobulated contour.

Homogeneous isointensity or mild hyperintensity on T1-weighted images and


heterogeneous high signal on T2-weighted or
short tau inversion recovery is seen.

For symptomatic patients, treatment should be based on the location of the


tumor and potential morbidity of treatment.

Treatment options include


- resection,
- radiation, or
- systemic therapy.

Radiation can be preoperative, intraoperative, or postoperative.

Systemic therapies for desmoids include


- anti-inflammatory drugs,
- hormonal (tamoxifen) agents,
- biologic agents (including the tyrosine kinase inhibitor imatinib), or
- chemotherapy (doxorubicin).

The decision making for treatment recommendations should be


multidisciplinary and should include the preferences and quality-of-life
considerations of the patient.

Carcinoid tumors originate in the intestine.

Frequently the lymph nodes in the mesentery are seen on scans; however,
their appearance is different, causing a sclerosing rather than infiltrative
appearance.
Carcinoids show arterial hyperenhancement and tethering of bowel loops,
whereas desmoids tend to show delayed hyperenhancement.

Lymphoma, while the most common solid neoplasm of the mesentery, is


usually associated with bulky adenopathy with preservation of fat around the
mesenteric vessels, something characterized as the “sandwich sign.”

Specifically, the nodes are dorsal and ventral to the mesenteric vessels,
appearing similar to “sandwich buns” with the mesenteric vessels and
surrounding fat appearing as the “sandwich meat.”

A leiomyoma or liposarcoma would show up as mostly fat (signal intensity


similar to subcutaneous fat, which can be used as a reference).

Figure 1. MRI showing 2 masses in the mesentery.


Figure 2. Flow chart illustrating the approach to mesenteric masses..
Table 1. Mesenteric Masses Summary With Discriminating Features at MRI.

==============================================================================

A 26-year-old woman with a history of cesarean section and familial


polyposis coli presents to clinic with an enlarging mass along her lower
abdomen. On exam, the mass is the size of a cantaloupe and involves much
of the lower abdominal wall. A core needle biopsy reveals monoclonal
fibroblasts appearing as spindle-shaped cells separated by an abundant
collagenous matrix. MRI reveals the mass extending into but not deep to the
left rectus muscle. First-line therapy for this lesion would be
A
surgical excision.
B
radiotherapy.
C
nonsteroidal anti-inflammatory drugs.
D
chemotherapy.
E
hormonal therapy.

A
surgical excision.

Desmoid tumors are benign mesenchymal tumors that can produce


significant morbidity and occasional mortality due to their multifocal and
locally infiltrative nature.

Their incidence ranges from 2.4–4.3 per million per year, and they are 2–3
times more common in women.

They often are sporadic in nature, resulting from a somatic mutation in the
beta-catenin gene.

In such instances, they typically do not have any apparent predisposing


factors.

A minority of desmoids, however, occur in the setting of familial


adenomatous polyposis (FAP) or in association with trauma or pregnancy.

Desmoids can arise from connective tissue from anywhere in the body, but
the abdominal wall, extremities/trunk, and intra-abdominal space are the
most common sites of presentation.

Symptoms range from nothing to bleeding, pain, and neuropathy.


MRI is the primary modality for imaging desmoid tumors because of its
ability to show the degree of tumor infiltration into surrounding structures.

Biopsy of these lesions reveal typical histologic features of monoclonal


fibroblasts appearing in spindle-shaped cells separated by abundant
collagenous matrix.

Surgical wide-margin excision remains first-line therapy for desmoids that


are resectable (figure 1 and figure 2).

Recurrence rates, however, can approach 40%, and preservation of function


must take precedent over clear margins when vital structures are involved.

Radiotherapy is indicated in patients with unresectable disease, positive


margins after surgery, or where resection would lead to unacceptable loss of
function.

In these settings, it can be given in both the neoadjuvant and adjuvant


setting.

When combined with surgery in patients who have positive margins, it can
improve local control rates in 75% of cases.

Nonsteroidal anti-inflammatory drugs (NSAIDs) decrease proliferation of


desmoids cells through the inhibition of COX-2, a cyclooxygenase
overexpressed in these tumors.

NSAIDS produce complete to partial responses and are indicated in patients


with unresectable or advanced disease without clinical symptoms.

Chemotherapy is indicated in cases of advanced disease with symptoms or


rapidly growing tumors unresponsive to other therapies.

Doxorubicin is effective at stabilizing growth, and doxorubicin-based


regimens, as monotherapy or in multiagent treatments, are commonly used.

Finally, antiestrogen drugs are effective in the treatment of desmoid tumors.

Tamoxifen is the most frequently used drug, and, like NSAIDs and
chemotherapy, it is often used in the setting of an unresectable tumor.

Figure 1. Surgical wide-margin excision remains first-line therapy for desmoids that
are resectable.
Figure 2. Surgical wide-margin excision remains first-line therapy for desmoids that
are resectable.

===============================================

A 52-year-old healthy man presents with 2–3 months of hematochezia.


Colonoscopy demonstrates a 5-cm mass 4 cm from the anal verge. Biopsy is
consistent with moderately differentiated adenocarcinoma. Endorectal
ultrasound shows tumor through the rectal muscular wall and an abnormal
1.5-cm mesorectal node (uT3N1). Which of the following statements is true?
A
Pathologic complete response is rare after neoadjuvant chemoradiotherapy.
B
Surgery may be avoided in complete responders to neoadjuvant therapy.
C
Total mesorectal excision alone results in a local recurrence rate of less than
5%.
D
Postoperative radiotherapy improves overall survival.
E
Postoperative chemoradiotherapy is preferable to preoperative
chemoradiotherapy.

B
Surgery may be avoided in complete responders to neoadjuvant therapy.

In locally advanced (T3 or N1-2) rectal adenocarcinoma identified on


endorectal ultrasound or cross-sectional imaging, preoperative
chemoradiotherapy is recommended to improve outcomes of treatment.

The addition of radiotherapy to systemic chemotherapy before definitive


surgery is beneficial in several ways compared with postoperative treatment:
- improved sphincter preservation rates,
- improved negative resection margin rate,
- lower toxicity, and
- higher compliance with treatment rates.

In addition, up to 20% of neoadjuvant therapy patients will have a complete


pathologic response to therapy, which is a strong predictor of long-term
survival.

In closely monitored patients with clinical complete response to neoadjuvant


therapy, an intense program of physical exam, imaging, and endoscopic
surveillance after clinical complete response is an acceptable option to
surgery and is associated with long-term, disease-free survival. 

In randomized trials comparing total mesorectal excision with or without


preoperative radiation, local recurrence rates were halved with the addition
of preoperative radiation to 5% or less.

Radiotherapy, although beneficial in improving local recurrence rates, does


not increase overall survival in trials examining modern surgical techniques.

===============================================

A 65-year-old man presents with increasing abdominal distention and a


retroperitoneal mass on CT scan (figure 1). CT is negative for lung or liver
metastases. An image-guided core biopsy shows well-differentiated
liposarcoma. Operative resection is planned. Which of the following
statements concerning his management is true?

[+] CLICK IMAGE TO ENLARGE


FIGURE 1

A
Resection is usually curative.
B
The addition of adjuvant radiation improves survival.
C
Adjuvant chemotherapy is recommended.
D
The kidney can be spared during operation.
E
Postoperative surveillance with imaging improves survival.

E
Postoperative surveillance with imaging improves survival.

Liposarcomas are the most common type of sarcoma.

They arise in the retroperitoneum, accounting for 41% of retroperitoneal


sarcomas (RPS); 12–40% of liposarcomas occur in the retroperitoneum
(figure 2).

Figure 2. Liposarcoma in the retroperitoneum

Cytogenetically, liposarcomas are characterized by


- supernumerary ring and
- giant rod chromosomes that contain amplifications of chromosome
12q.

Most retroperitoneal liposarcomas are well-differentiated or de-differentiated


tumors; the former have a tendency to recur locally but have a low
metastatic potential.

Approximately one-quarter of well-differentiated tumors transform into de-


differentiated tumors with a worse prognosis.

The average age at diagnosis is 60–70 years.


Because of the potential spaces in the retroperitoneum, these tumors can
grow quite large before they cause vague symptoms; half are larger than 20
cm when diagnosed.

Preoperative biopsy is controversial—most fatty masses in the


retroperitoneum are malignant, and biopsy is sometimes reserved for cases
in which neoadjuvant therapy is considered.

Imaging clues that a liposarcoma may be higher grade include


- dense areas,
- septations,
- heterogeneity,
- enhancement, and
- irregular margins.

The major prognostic factors for RPS are grade and completeness of
resection; therefore, multifocality and invasion of adjacent structures are
associated with worse outcome.

Stage is less important because, by definition, all RPS are deep, and most
are greater than 5 cm, making these tumors at least stage IIB or III at
diagnosis.

Complete resection often involves surrounding organs such as kidneys,


adrenals, or segments of intestine, especially if they are encased in tumor,
as is the case with this patient’s right kidney.

Radiation therapy, given either preoperatively or postoperatively, can reduce


local recurrences and lengthen recurrence-free survival.

Few data show an effect on actual survival.


The role of adjuvant or neoadjuvant chemotherapy remains controversial.

Even after aggressive local control, local recurrence is common; 5-year local
recurrence rates are 50% for well-differentiated and 80% for de-
differentiated retroperitoneal liposarcomas.

Recurrences diagnosed and resected early experience complete resection


rates of 90% and the same 5-year survival rates as the primary resection.

Most recurrences are asymptomatic; therefore, close imaging follow-up,


especially of the resection beds, improves survival.
This close imaging follow-up is part of the National Comprehensive Cancer
Network guidelines for this tumor.

Outcomes after reresection are best with longer disease-free intervals and
negative specimen margins.

===============================================

A 72-year-old woman presents with increasing abdominal girth. She has a


CT scan that is shown in figure 1. The ascites is tapped and described as
gelatinous. No malignant cells are seen on cytology. An intraoperative
photograph is shown in figure 2. Final pathology demonstrates abundant
extracellular mucin with scant cells and no atypia. The most common cause
for this disease is

[+] CLICK IMAGE TO ENLARGE


FIGURE 1

[+] CLICK IMAGE TO ENLARGE


FIGURE 2

A
mucinous cystadenoma of the appendix.
B
mucinous cystadenocarcinoma of the appendix.
C
mucinous neoplasm of the ovary.
D
signet cell colon cancer.
E
primary peritoneal carcinoma.
A
mucinous cystadenoma of the appendix.

Pseudomyxoma peritonei (PMP) is a clinical entity that continues to be


confusing and poorly understood.

This confusion is partly related to the heterogeneity of the disease and the
significantly different outcomes associated with this heterogeneity.

Clinically, PMP often presents in an indolent fashion.

Patients may note an increase in abdominal girth and general malaise.

A CT scan shows diffuse “ascites.”

However, if tapped, the fluid is thick and gelatinous, consisting of mucin.

At surgery, this material is similar in texture to jelly; hence, the term “jelly
belly” is used.

The common factor in the literature is the abundance of mucin.

To better clarify PMP, 2 clinical entities were described:


- disseminated peritoneal adenomucinosis (DPAM) and
- peritoneal mucinous carcinomatosis (PMCA).

This distinction is helpful, because the natural history of these 2 entities


seems quite different.

DPAM is characterized by
abundant gelatinous material that
involves only the peritoneal surface.

The associated epithelium is scant and difficult to find.

Histologically, the epithelium is considered “bland” and


has minimal cytologic atypia and rare mitotic activity.

The appendix is the most likely source and the spread is attributed to
rupture of a benign cystadenoma of the appendix itself.

Parenchymal organ invasion is uncommon with DPAM,


but invasion of the ovary is the most common presentation.

The natural history of DPAM is indolent.

Recurrence of the gelatinous material is not uncommon, and surgical


debulking is often necessary both for palliation and to treat associated
complications such as bowel obstruction.

Death can occur but is less common than PMCA cases and usually occurs
after many years.

The 5-year survival rate for DPAM is 84%.

By contrast, patients with PMCA have a far more aggressive course.

These patients have peritoneal lesions that, when biopsied, show


abundant epithelial tissue with marked atypia,
including signet ring cells, and
pathological features consistent with invasive adenocarcinoma,
including invasion of parenchymal tissue, such as the bowel wall.

Unfortunately, patients with these pathological features have a poor


prognosis, with a 5-year survival rate of 6.7%.

Most die within 3 years.

For patients with intermediate features, the 5-year survival rate is 37.6%.

Recent literature has altered the terminology somewhat.

Because disease recurrence is common, many practitioners still consider


DPAM more consistent with a true adenocarcinoma.

Therefore, the term mucinous adenocarcinoma low grade is now often advocated


and synonymous with DPAM,
whereas PMCA is now often termed mucinous adenocarcinoma high grade. 

Patients with low-grade disease have a better prognosis than those


associated with high-grade lesions.

The site of origin of PMP is also somewhat controversial.

Originally, it was suggested that the site of origin was the appendix (figure
3), the ovary, the peritoneum, or some combination these.
However, now the disease is thought to be associated with the appendix;
even when the ovary is involved, it is probably secondarily involved.

The appendix is still the most likely original source.

This belief is based on clinical observations and immunohistochemistry


studies that suggest these tumors are more likely to be of a gastrointestinal
origin.

Even in cases with ovarian involvement, no cases are of unequivocal ovarian


origin.

When the appendix appears to be normal, on very close histologic


evaluation, a mucinous cyst adenoma is often found.

Although there are mucinous-producing ovarian cancers, cases presenting


with PMP appear more likely to be of appendiceal origin.

In a 1995 series, 87% of the 109 cases likely came from the appendix or
colon, and no cases were unequivocally of ovarian origin.

Several of the cases classified as PMCA appeared to be mucinous producing


tumors of the colon, and the clinical presentation was consistent with
carcinomatosis.

The optimal treatment remains somewhat controversial.

Surgical debulking is the mainstay of treatment, but recurrence is common.

Some surgeons also advocate the use of intraperitoneal chemotherapy in


addition to aggressive surgical debulking.

The difficulty in interpreting outcomes is that many studies are relatively


small and there is often heterogeneity in reporting.

The aggressive approach involves extensive debulking of all visible tumors


and the use of intraperitoneal chemotherapy.
Figure 3. Figure 1. Figure 2.

===============================================

A 34-year-old G2P2 woman presented to her primary care provider with a


painful, slowly enlarging mass in her right lower quadrant. Three years ago,
she had a similar mass removed from the same location. The new mass
appears to be arising within the scar, and imaging confirms that the solid 5-
cm mass involves the rectus and oblique muscles without extension through
the peritoneum. Needle biopsy suggests a fibrous sarcoma. Which of the
following approaches would you recommend?
A
Wide excision with 3-cm margins
B
Resection to macroscopically clear margins followed by radiation therapy
C
Radiotherapy to 50 Gy
D
Chemotherapy followed by excision with 3-cm margins
E
Observation

B
Resection to macroscopically clear margins followed by radiation therapy

Desmoid tumors are


- slow-growing fibroblastic proliferative processes that
- are poorly circumscribed and locally
- invasive but lack metastatic potential.

Sporadic desmoid tumors often arise in postpartum women within 1–2 years
of pregnancy.

Cases not associated with pregnancy may involve the extremities and trunk
musculature, head and neck, and abdominal cavity.

Although spontaneous regression is reported, the mainstay of treatment is


surgical with a focus on preservation of function. 

Margin status after resection remains an independent prognostic indicator


for local recurrence.

However, wide resection beyond macroscopically clear margins offers no


additional benefit and may complicate closure of the abdominal wall.

Studies suggest that the addition of adjuvant radiation therapy results in


significantly better local control compared with surgery alone, although
randomized trials are limited due to the rarity of the disease.

In-field recurrence occurs mainly when the total irradiation dose is less than
50 Gy, while high rates of radiation-related complication are associated with
doses greater than 56 Gy. 

For patients with desmoid tumors not amenable to surgery or radiotherapy,


various treatments, including NSAIDS and traditional cytotoxic
chemotherapies have been tried but none have been subjected to controlled
clinical trial.

Anti-hormonal therapy is suggested, given the natural history of the disease


and its association with pregnancy.

Responses to high-dose tamoxifen are reported, although there are no


randomized confirmatory data.

In cases of unresectable, rapidly growing, or life-threatening desmoid tumor,


traditional cytotoxic chemotherapy (methotrexate and vinblastine or
vinorelbine) may be the treatment of choice, although the regimen is quite
toxic over time.

Targeted therapies, such as imatinib, have shown promise in patients in


whose tumors include genetic targets such as KIT, ABL, ARG, and
PDGFAR/B.
However, when possible, surgical resection remains the treatment of choice
for desmoid tumors.

===============================================

Which of the following is true regarding neutropenic enterocolitis?


A
It is seen in patients receiving cytotoxic agents.
B
Clostridium difficile is the causative organism.
C
Hematopoietic colony-stimulating factor treatment is contraindicated.
D
Early ileocecectomy decreases mortality.
E
Colonoscopy is required for diagnosis.

A
It is seen in patients receiving cytotoxic agents.

Neutropenic enterocolitis (NEC) or typhlitis usually involves the cecum,


which will have a thickened bowel wall that is edematous with varying
degrees of ulceration, ecchymosis, and hemorrhage.

NEC was most commonly seen in patients treated for acute leukemia, but it
is associated with many other conditions, including
- lymphoma,
- solid tumors,
- AIDS, and
- aplastic anemia or
- cyclic neutropenia who have not received cytotoxic agents.

There appears to be an association between NEC when taxanes and


vinorelbine are being used for cancer treatment.

Other agents associated with NEC include 5 fluorouracil, capecitabine,


cyclophosphamide, ifosfamide, cisplatin, and carboplatin.
The pathogenesis of NEC is multifactorial.
Neutropenia itself is a significant contributing factor.

Neutropenia clearly reduces the immune response against invasion of local


tissue by gastrointestinal microbes.

If neutropenia is severe, and treatment response is initially poor,


normalization of neutrophil counts can occasionally be achieved through the
administration of colony stimulating factors.

Otherwise, care is supportive and includes bowel rest, intravenous fluids,


parenteral nutrition, and broad-spectrum antibiotics 

NEC is frequently polymicrobial with multiple bacterial species and


occasionally fungi having a pathogenic role. 

Clostridium difficile associated colitis is a separate entity and should be


excluded.
Colonoscopy is not indicated nor is cecostomy.

Early ileocecectomy is generally not indicated and does not decrease


mortality.

==============================================

A 51-year-old woman has a laparoscopic right colectomy for a right colon


cancer. She has no evidence of metastatic disease on her preoperative
evaluation. Her final pathology is consistent with a stage II colon cancer.
Which of the following factors is associated with an improved prognosis? 
A
Poor differentiation
B
T4 tumor
C
Lymph node harvest less than 12
D
Microsatellite instability
E
Preoperative obstruction
D
Microsatellite instability

The management of stage II colon cancer remains a bit controversial.

Chemotherapy trials have not shown clear advantages for routine


administration of chemotherapy to all patients with stage II colon cancer, so
it is not routinely recommended.

However, approximately 20–25% of patients with stage II disease will recur


with an overall survival of approximately 80%.
Therefore, there are attempts to stratify stage II patients into high- and low-
risk groups.

For the high-risk patients, a discussion of adjuvant chemotherapy may be


relevant. 

The following are all associated with poor outcomes for patients with stage II
colon cancer:
- Poorly differentiated tumors,
- T4 tumors,
- lymph node harvest of less than 12 nodes, and
- preoperative obstruction

These patients should be considered at higher risk for systemic failure and
may be considered for chemotherapy.

It is important, therefore, that these patients get referrals to medical


oncologists to discuss the pros and cons of chemotherapy under these
circumstances. 

By contrast, microsatellite instability appears to be associated with improved


overall survival.

In addition, tumors that have a high level of microsatellite instability seem


resistant to traditional chemotherapy for colon cancer (5-fluorouracil-based
therapy), suggesting that chemotherapy is unlikely to offer benefit to this
subset of patients. 

The decision to offer systemic chemotherapy to stage II colon cancer


patients is therefore complex.
It seems logical that some high-risk stage II patients would benefit from
therapy, but it is important to understand that that overall benefit is likely to
be small (<5%) and must be considered when patients and physicians
contemplate treatment.

Surgeons, however, need to recognize poor prognostics features so that


proper referrals can be made.

===============================================

A 60-year-old man presents with dysphagia. Upper endoscopy with


ultrasound reveals a distal esophageal mass invading into the esophageal
adventitia and regional lymphadenopathy. Endoscopic biopsies of the
esophageal mass and an enlarged regional lymph node confirm esophageal
adenocarcinoma. Whole-body PET scan shows the distal esophageal mass
and regional lymphadenopathy but no distant metastases. Regarding
preoperative therapy for esophageal adenocarcinoma,
A
the addition of preoperative radiation to chemotherapy significantly improves
overall survival.
B
preoperative chemoradiation therapy increases postoperative mortality.
C
tumor response to preoperative chemotherapy is a prognostic factor for
overall survival.
D
preoperative chemotherapy does not affect local recurrence rates.
E
esophageal perforation is the primary cause for failure to complete planned
preoperative chemoradiation therapy.

C
tumor response to preoperative chemotherapy is a prognostic factor for
overall survival.
The most common treatment for resectable esophageal adenocarcinoma is
preoperative chemoradiation therapy followed by surgical resection,
although this approach is still under active investigation.

Preoperative chemotherapy alone or with radiation, improves overall survival


compared with surgical resection alone.

A large study of patients receiving neoadjuvant chemotherapy showed that


only patients who were “downstaged” had a decreased incidence of local and
systemic recurrence as well as an increase in their overall survival. 

The addition of preoperative radiation to chemotherapy does not definitively


improve overall survival over chemotherapy alone.

A meta-analysis showed that neither preoperative chemotherapy nor


preoperative chemoradiation therapy increased the risk of postoperative
complications compared with surgery alone. 

Esophageal perforation during preoperative chemoradiation therapy is a rare


event, occurring in 1 of 171 patients recently studied.

Fatigue, leukopenia, thrombocytopenia, and nausea are the most common


side effects of preoperative chemoradiation treatment, occurring in 50% or
more of patients.

===============================================

Which of the following treatment strategies for adjuvant therapy after


surgical resection of gastric adenocarcinoma is effective in randomized
controlled trials?
A
Intravenous 5-fluorouracil, leucovorin, and external beam irradiation
B
Intravenous epirubicin and cisplatin chemotherapy
C
Oral capecitabine
D
Intravenous oxaliplatin, interferon, and external beam irradiation
E
Oral imatinib
A
Intravenous 5-fluorouracil, leucovorin, and external beam irradiation

Although complete resection of cancer with lymph node dissection is the only
curative treatment for gastric cancer, a high rate of locoregional as well as
distant recurrences are reported.

Local and regional failure after surgical treatment of gastric carcinoma


occurs in 40–65% of patients and nearly always results in cancer-related
death.

Five-year survival for these patients is approximately 20%. A survival


benefit is obtained from the addition of chemotherapy or chemoradiotherapy
to surgery alone, whereas no benefit is obtained with adjuvant radiotherapy
alone. 

In the last few decades, several randomized controlled trials investigated the
role of adjuvant therapy versus surgery alone.

The Intergroup trial (INT 0116) demonstrated improved outcomes for


patients who receive an R0 (negative margin) surgical resection and
postoperative chemotherapy (bolus fluorouracil) and radiation therapy (45
Gy delivered 2 cm beyond all involved nodal basin).

In the CLASSIC phase III trial, 1035 patients with stage II–IIIB gastric
cancer were randomly assigned to receive surgery followed by chemotherapy
with oral capecitabine and oxaliplatin (XELOX) or surgery alone.

This study demonstrated a 34% reduction in the risk of death in the XELOX
arm.

Oral capecitabine was not tested alone, and interferon was not added to this
regimen in a randomized controlled trial.

The MAGIC trial tested the survival benefit of perioperative (before and after
surgery) chemotherapy and surgery versus surgery alone.

Chemotherapy consisted of 3 preoperative and 3 postoperative cycles of


intravenous epirubicin, cisplatin, and fluorouracil.
Compared with the surgery group, the perioperative-chemotherapy group
had a higher likelihood of overall survival and progression-free survival.

Together, the results of these randomized controlled trials suggest that


additional treatment including systemic chemotherapy is beneficial to
patients with gastric cancer who receive surgical resection.

Although several answers list agents found in some of the published trials,
only intravenous 5-fluorouracil, leucovorin, and external beam irradiation
covers all of the reagents and modalities in the same manner as that used in
a randomized control trial demonstrating survival benefit. 

Oral imatinib is an agent with activity against gastrointestinal stromal


tumors but not adenocarcinoma.

===============================================

A 69-year-old man presents with hematemesis. He is anemic and has a 3-


month history of weight loss and fatigue. Upper endoscopy reveals a large
gastric ulcer in his antrum that is positive for adenocarcinoma on biopsy. CT-
guided biopsy reveals evidence of a liver metastasis. Which of the following
tumor markers will affect management?
A
HER2
B
VEGF
C
EGFR
D
mTOR
E
c-MET

A
HER2
Gastric cancer is the second leading cause of cancer death worldwide, in part
because it often presents at an advanced stage.

In Western developed countries, outcomes remain poor, with 5-year


survivals hovering around 50% in Europe.

Treatment of advanced-stage disease using epirubicin-based variants of the


original epirubicin, cisplatin, and fluorouracil (ECF) chemotherapy regimen
demonstrate overall survival and quality-of-life benefits comparable with
traditional regimens.

More recently, clinical trials focused on the role of targeted biologic therapy
using monoclonal antibodies/inhibitors to overexpressed proteins in treating
advanced-stage cancers.

Of these, trastuzumab, the antibody to human epidermal growth factor


receptor 2 (HER2), demonstrates an overall survival benefit and is now
recommended therapy in individuals with HER2-positive tumors.

HER2 positivity is seen more frequently in


- intestinal (34%) and
- esophagogastric junction (32%) subtypes of gastric cancer, but it can
be found in
- the diffuse-type (6%) and
- purely gastric (18%) variants.

It is associated with tumor invasion, high-grade histology, and poor


prognosis.

Vascular endothelial growth factor (VEGF) is overexpressed in up to 60% of


tumors and is associated with
- advanced stage,
- tumor aggressiveness, and
- increased rates of recurrence.

The AVAGAST trial using the anti-VEGF antibody bevacizumab did not
demonstrate improved overall survival with its use in combination with
chemotherapy compared with placebo.

Like VEGF, epidermal growth factor receptor 1 (EGFR) is overexpressed in


60% of gastric cancers and is associated with
- advanced stage and
- poor prognosis.
Like bevacizumab, the anti-EGFR antibody cetuximab does not demonstrate
a benefit in overall survival.

The mammalian target of rapamycin (mTOR) is a serine/threonine kinase.

Its inhibitor, everolimus, does not show benefit to overall survival.

Finally, mesenchymal-epithelial transition factor (c-MET) overexpression is


associated with
- poor prognosis in gastric cancer;
the effectiveness of its antibody onartuzumab is currently undergoing
investigation.

In this patient with stage IV gastric cancer, HER2 positivity would prompt
treatment with trastuzumab.

===============================================

A 62-year-old woman has a colonoscopy for microcytic anemia. A cecal


cancer is identified and confirmed by biopsy. Three years earlier, she had a
colonoscopy that found an 8-mm serrated sessile adenoma at the hepatic
flexure that was completely removed. She has no family history of colon
cancer. The etiology of the cecal tumor is most likely associated with
A
chromosomal instability of the APC gene.
B
defect in mismatch repair genes.
C
BRAF wild type.
D
hypermethylation.
E
kRAS mutation.

D
hypermethylation.
Until recently, 2 primary pathways were described for the development of
colorectal cancer.

The adenoma-to-carcinoma sequence results from a series of genetic


defects, beginning with a defect in the APC gene, a tumor suppressor gene.

Subsequent genetic defects result in predictable histologic changes that


begin as an adenomatous polyp and progress to an invasive
adenocarcinoma.

This pathway accounts for the majority of sporadic colon cancer and is
frequently described as the chromosomal instability pathway. 

The second genetic pathway results from a defect in mismatch repair genes.

This defect is associated with microsatellite instability and is the genetic


defect responsible for Lynch Syndrome, which accounts for 3–5% of all colon
and rectal cancers.

These tumors are more likely to be


- proximal and have
- mucinous histology, a
- Crohn-like lymphocytic infiltration, and
- poorly differentiated histology.

Despite this, overall survival is more favorable compared with colon cancer
associated with other genetic aberrations. 

More recently, an additional pathway resulting in colon cancer was


described.

This is an epigenetic pathway involving hypermethylation CpG islands.

The CpG island methylator phenotype (CIMP) pathway is responsible for


sessile-serrated polyps.

These polyps, previously considered to be hyperplastic and not


premalignant, are now known to be a cause of colon cancer.

These sessile serrated polyps can be very difficult to detect endoscopically,


because they tend to be quite flat and subtle.

Furthermore, the time to progress from polyp to cancer may be faster than
adenomatous polyps.
This may be one explanation for patients who develop colon cancer despite a
recent normal colonoscopy (interval colon cancer).

In a 2010 publication, patients with interval colon cancers were 2.5 times
more likely to be CIMP+ compared with noninterval cancers, indicating that
many of these tumors may have originated as sessile serrated polyps.

CIMP is associated with BRAF mutations not wild type BRAF.

KRAS mutations are not associated with tumors harboring CIMP.

In the clinical scenario, this patient has developed a new colon cancer
despite having had a colonoscopy 3 years earlier.
At that colonoscopy, she did have a sessile serrated polyp.
Given the history of the serrated adenoma and the rapid development of an
interval cancer, this presentation is very consistent with a tumor developing
through this CIMP pathway.

Therefore, hypermethylation is the best answer.

===============================================

The use of cytoreductive surgery and hyperthermic intraperitoneal


chemotherapy (HIPEC) for peritoneal carcinomatosis from colorectal cancer
A
is contraindicated in the presence of liver metastases.
B
is more effective than HIPEC performed for pseudomyxoma peritonei.
C
is supported by prospective randomized data.
D
is not associated with survival benefit.
E
is unaffected by the extent of disease.

C
is supported by prospective randomized data.
Peritoneal carcinomatosis from colorectal cancer occurs in 30–40% of
patients and is a particularly fatal form of metastatic disease.

In the past, this form of disease progression was considered terminal, with a
6-month median survival.

Median survival is 12–24 months with palliative combination chemotherapy. 

Selected patients with limited peritoneal carcinomatosis may be able to


undergo cytoreductive surgery.

Survival is obviously better when the burden of peritoneal carcinomatosis is


limited.

The prognosis of peritoneal carcinomatosis is worse when it is not


resectable.

Studies from Japan, Germany, and other countries demonstrated that


complete resection of peritoneal carcinomatosis can afford a survival benefit
with or without the addition of hyperthermic intraperitoneal chemotherapy
(HIPEC). 

Although the combination of cytoreductive surgery and HIPEC is clearly most


effective for patients with lower grade tumors such as pseudomyxoma
peritonei, in colorectal peritoneal carcinomatosis, the combination of
cytoreductive surgery and HIPEC has produced cure rates of up to 16% in
prospective studies.

In fact, colorectal cancer is the only form of peritoneal carcinomatosis for


which cytoreductive surgery and HIPEC was subjected to a randomized trial.

A randomized controlled study of chemotherapy alone versus cytoreductive


surgery and HIPEC for colorectal peritoneal carcinomatosis showed a median
survival 2-fold higher for the cytoreductive surgery and HIPEC arm, but only
38% of patients were completely resectable.

“Resectable” patients in this and other studies included


- patients with up to 4 liver metastases;
- thus, hepatic metastases are not a contraindication to cytoreductive
surgery and HIPEC combined therapy.

===============================================
A 62-year-old woman is diagnosed with a rectal cancer 7 cm from the anal
verge. On physical exam, the lesion is posteriorly located and mobile. A
pelvic MRI demonstrates a 3-cm tumor that extends 5 mm into the
mesorectum. No lymphadenopathy is noted. Biopsies confirm a moderately
differentiated adenocarcinoma. A chest/abdomen/pelvis CT scan is negative
for metastatic disease. The next best step in management is
A
local excision.
B
chemoradiation.
C
radiation.
D
low anterior resection.
E
abdominal perineal resection.

B
chemoradiation.

The management of rectal cancer has evolved over the past 30 years.

Historically, recurrence rates, especially local recurrence rates, were very


high.

Treatments were aimed at methods to minimize these poor outcomes.

Over the years, a series of trials examined radiation, chemoradiation, and


surgical technique to optimize rectal cancer treatment.

With these studies, it became clear that multimodality treatment obtained


the best results.

The remaining questions revolved around the timing of the adjuvant


therapy.
The German Rectal Cancer Trial, first published in 2004 and updated in
2012, examined whether chemoradiation should be given before surgery or
postoperatively.

This study concluded that neoadjuvant chemoradiation significantly reduced


the risk of local recurrence compared with postoperative chemoradiation
(7% vs 10%).

Part of this effect may be that preoperative therapy is less toxic, and
compliance with completing the chemoradiation was higher in the
neoadjuvant arm.

Nevertheless, this trial seemed to answer the key question of timing, and
neoadjuvant chemoradiation emerged as the clear choice for locally
advanced rectal cancer. 

Current National Comprehensive Cancer Network guidelines recommend


neoadjuvant chemoradiation for all stage II and III mid-rectal cancers.

Clinical staging of rectal cancer is critical to management decisions.

High-definition MRI is evolving as the preferred method to locally stage


rectal cancer.

With MRI, the depth of invasion into the mesorectum and an assessment of
the mesorectal margin can be more precise.

In this case, the tumor does extend 5 mm into the mesorectum, making this
a clear T3 cancer.
No lymphadenopathy is noted, and a CT scan shows no evidence of distant
disease.
Clinically, this tumor is a T3N0M0, stage II mid-rectal cancer.

Neoadjuvant chemoradiation is therefore indicated to improve local control,


promote tumor downstaging, and improve chances of sphincter salvage. 

Surgery should not be done as the initial treatment for this tumor.

Furthermore, under most circumstances, an abdominal perineal resection


would be unnecessary for a tumor at 7 cm from the anal verge.

Local excision is now limited to very early rectal cancers (T1) and would not
be appropriate therapy for a T3 cancer with curative intent.
Radiotherapy alone is less effective in tumor downstaging and is not
generally used.
It is used for short-course radiotherapy, which is more popular in Europe but
rarely used in the United States.

===============================================

A 50-year-old man undergoes esophagogastroduodenoscopy (EGD) for


symptoms of vague intermittent abdominal pain. Areas of antral gastritis are
noted. Biopsies obtained show the presence of Helicobacter pylorias well as
low-grade, mucosa-associated lymphoid tissue (MALT) lymphoma. The most
appropriate initial treatment is
A
observation with repeat EGD in 6 months.
B
H. pylori eradication.
C
radiation.
D
chemotherapy.
E
subtotal gastrectomy.

B
H. pylori eradication.

Mucosa-associated lymphoid tissue (MALT) is acquired secondary to chronic


gastritis from Helicobacter pylori infection.

Although only a small percentage of patients with H. pylori infection


eventually develop MALT, nearly all patients with MALT are H. pylori positive. 

H. pylori eradication is the initial treatment of choice in low-grade MALT, with


more than 75% of patients achieving complete regression of disease after
initial treatment.

High-grade transformation into invasive lymphoma is extremely rare


(<1%). 
Observation with repeat esophagogastroduodenoscopy is not appropriate,
because H. pylori eradication has proven to be successful.

This watch-and-wait strategy is reserved for those with histological residuals


of MALT after initial therapy and normalization of endoscopic abnormalities.

Radiation and chemotherapy are options for patients who do not respond to
initial H. pyloritreatment and who have high-grade gastric lymphoma.

Surgery is never indicated as the initial treatment for MALT, as it is not


superior to H. pylori eradication.

===============================================

32-year-old man has a clinical and radiographic diagnosis of appendicitis.


The pathologic analysis of the resected appendix notes an appendiceal
neuroendocrine (carcinoid) tumor. Which of the following histopathologic
characteristics of the tumor would mandate further therapy after simple
appendectomy?
A
Presence of 0–1 mitoses/10 high-powered fields
B
No mesoappendiceal involvement
C
Goblet cell histology
D
1.5-cm tumor size 
E
Perforation of the appendix

C
Goblet cell histology

Appendiceal neuroendocrine tumors, or “carcinoids,” are usually indolent


tumors with a low propensity for recurrence.
If incidentally found at time of appendectomy, the histologic features of the
tumor and findings at the index operation dictate the need for further
therapy.

Neuroendocrine tumors are associated with regional lymph node metastases


in a linear fashion based on the size of the tumor.

Tumors less than or equal to 1 cm have a low incidence of lymph node


metastasis and excellent long-term survival.

Tumors greater than 2 cm in size have a frequent (>40%) incidence of nodal


metastasis, and right colectomy would be indicated for assessment of
regional nodes.

There is a lower, but definite, incidence of nodal metastases in tumors from


1–2 cm in size, and other factors must be considered to determine whether
further surgical intervention would be warranted.

The following tumors have no added risk


- low mitotic rate (<2 mitoses/10 HPF),
- without mesoappendiceal involvement, and
- with perforation of the appendix. 

Goblet cell histology indicates an adenocarcinoid, which shares features of


adenocarcinoma and carcinoid.
Goblet cell histology is associated with much poorer outcomes compared
with pure neuroendocrine tumors.

Unless very small, Goblet cell tumors of the appendix should be managed
with a right hemicolectomy.

===============================================
 
The most common primary cancer of the small intestine is
A
lymphoma.
B
adenocarcinoma.
C
neuroendocrine tumors.
D
gastrointestinal stromal tumor.
E
melanoma.

C
neuroendocrine tumors.

Historically, small bowel adenocarcinomas were considered the most


common primary malignancy of the small intestine.

The incidence of neuroendocrine tumors of the small intestine (carcinoid


tumors) has increased by more than 400% in the last several decades for
unclear reasons.

In a review of the National Cancer Data Base and the Surveillance


Epidemiology and End Results databases, more than 38,000 cases of small
bowel tumors were identified.

Neuroendocrine tumors have replaced small intestinal adenocarcinomas as


the most common primary small bowel cancer.

Although the rapid increase in incidence of neuroendocrine tumors of the


small bowel has been observed, the relative incidence of small bowel
adenocarcinoma, gastrointestinal stromal tumors, and lymphomas remained
stable over time.

Metastatic melanoma is the most common secondary malignancy of the


small bowel but is not a primary intestinal tumor.

===============================================

A tyrosine kinase inhibitor effective in the treatment of medullary thyroid


cancer
A
Anastrozole
B
Cabozantinib
C
Everolimus
D
Letrozole
E
Sunitinib

B
Cabozantinib

An aromatase inhibitor effective in the prevention of breast cancer in


postmenopausal women at high risk of developing breast cancer

A
Anastrozole

A tyrosine kinase inhibitor effective in the treatment of advanced


gastrointestinal stromal cell tumors

E
Sunitinib

A mammalian target of rapamycin (mTOR) inhibitor that prolongs


progression-free survival in patients with advanced pancreatic
neuroendocrine cancer

C
Everolimus

New pharmacologic agents continue to be developed and offered as


treatment for patients with cancer.

Anastrazole is a member of the family of agents classified as aromatase


inhibitors.
Aromatase inhibitors inhibit the endogenous conversion of androgens to
estrogens.

Anastrazole is US Food and Drug Administration (FDA) approved for use as


adjuvant treatment or first-line treatment of locally advanced or metastatic
breast cancer in hormonally sensitive tumors.

In addition, in a recent double-blind, randomized, placebo-controlled trial,


anastrazole was shown to substantially reduce the incidence of breast cancer
in high-risk postmenopausal women. 

Letrozole is an oral nonsteroidal aromatase inhibitor used for the treatment


of hormonally responsive breast cancer after surgery.

To date there are no currently available published data supporting the use of
this drug as a method of breast cancer risk reduction.

Cabozantinib is a tyrosine kinase inhibitor that is FDA approved for the


treatment of medullary thyroid cancer (MTC).

Although complete surgical resection is curative for some patients with MTC,
patients with distant metastases may have a shorter median survival time.

Cabozantinib is associated with an improvement in estimated progression-


free survival compared with placebo in patients with progressive MTC. 

Sunitinib is an oral, small-molecule, multitargeted receptor tyrosine kinase


inhibitor that is FDA approved for the treatment of imatinib-resistant
gastrointestinal stromal tumors (GISTs).

A randomized, double-blind, placebo-controlled trial showed that patients


with unresectable GISTs initially unresponsive to imatinib who were
subsequently treated with sunitinib achieved significantly greater
progression-free and overall survival.

Everolimus is a derivative of sirolimus; both are mammalian target of


rapamycin (mTOR) inhibitors.

In a double-blind, phase 3 study, patients with advanced, low-grade or


intermediate grade pancreatic neuroendocrine tumors with radiologic
progression within the previous 12 months were randomly assigned to
treatment with either oral everolimus or placebo.
Patients treated with everolimus realized significantly prolonged progression-
free survival.

===============================================

First-line chemotherapeutic agent for gastric cancer only


A
5-fluorouracil
B
Oxaliplatin
C
Trastuzumab
D
Bevacizumab
E
Epirubicin

E
Epirubicin

Anti-HER2 agent

C
Trastuzumab

Platinum-based agent effective in colon cancer

B
Oxaliplatin

Much progress has recently occurred related to therapeutic options for the
treatment of gastrointestinal cancers.
Although 5-fluorouracil (5-FU) remains a staple of first-line
chemotherapeutic regimens for both colorectal and gastric cancers, other
biologic and chemotherapeutic agents are as effective in gastrointestinal
cancer treatment. 

Epirubicin, a topoisomerase inhibitor, is an effective agent for treating


gastric cancer in combination with cisplatin and 5-FU.
It was also found to be effective with capecitabine and oxaliplatin.

Epirubicin-based regimens now serve as the first-line chemotherapeutic


intervention for gastric cancer. 

Trastuzumab, a monoclonal antibody to the human epidermal growth factor


receptor 2 (HER2), demonstrates improved overall survival when given in
combination with chemotherapy for advanced gastric cancer with HER2
positivity. 

Oxaliplatin, a platinum-based chemotherapeutic agent, demonstrates


synergistic effects with 5-FU in the treatment of colon cancer, resulting in
better response rates and time to disease progression in patients with
advanced colon cancer. 

Finally, bevacizumab is an antivascular endothelial growth factor agent that,


when added to 5-FU, leucovorin, and oxaliplatin (FOLFOX) in the treatment
of advanced colon cancer, leads to improved response rates, time to disease
progression, and overall survival compared with FOLFOX alone.

===============================================

One-third will require surgical intervention. 


A
Acute radiation enteritis
B
Chronic radiation enteritis
C
Both
D
Neither
B
Chronic radiation enteritis

Symptoms often resolve with nonoperative management.

A
Acute radiation enteritis

Severe malnutrition can occur, requiring parenteral nutrition.

C
Both

Enteroclysis is the most accurate imaging test for diagnosis.

B
Chronic radiation enteritis

Radiation injury causes cell death via apoptosis from free radical generation.

Radiation has its greatest effect on rapidly proliferating cells, making the
small intestine epithelium particularly susceptible to radiation injury.

Most cases occur with doses greater than 4500 cGy.

Acute radiation enteritis is a transient entity, occurring in up to 75% of


patients who undergo abdominal and pelvic radiation.

Chronic radiation enteritis will develop in 5–15% of these patients. 

Acute radiation injury to the small intestine results in a decline in crypt cell
mitosis and mucosal cell necrosis, which leads to villous sloughing.

Because only the mucosal layer is involved, cessation of radiation therapy


results in a resolution of symptoms.
Symptoms most commonly include diarrhea, vomiting, nausea, and crampy
abdominal pain.

The diagnosis is made from the clinical history, and additional testing is not
required.

Most cases are self-limited and resolve with conservative management.


Patients with severe diarrhea or vomiting may require hospital admission,
fluid resuscitation, and parenteral nutrition. 

Chronic radiation injury is caused by a progressive occlusive vasculitis,


leading to chronic ischemia and fibrosis and affecting all layers of the
intestinal wall.

This can lead to strictures, abscesses, and fistulae.

Enteroclysis is the most accurate diagnostic test for chronic radiation


enteritis, with sensitivity reported as high as 90%.

Treatment for chronic radiation enteritis is difficult.

Parenteral nutrition is often required, and up to one-third of patients


eventually require surgical intervention, usually for high-grade obstruction,
perforation, hemorrhage, or fistulae.

===============================================

Retrieving 12 lymph nodes is a quality metric. 


A
Colon cancer
B
Rectal cancer
C
Both
D
Neither

C
Both
Circumferential radial margin has prognostic value.

C
Both

Neoadjuvant chemoradiation therapy is used for stage II disease

B
Rectal cancer

Bevacizumab prolongs overall survival in the adjuvant setting.

D
Neither

Oncologic outcomes are equivalent with the laparoscopic approach


compared with the open approach.

A
Colon cancer

Colorectal cancer is currently the third most common cancer and the third
leading cause of cancer death for women and men in the United States.

In 2014, more than 136,000 new cases of colorectal cancer were diagnosed.

Although there are similarities between colon cancer and rectal cancer, there
are differences.

Both colon and rectal cancer use a


- similar staging system, and
- surgical resection offers the best chance for a cure for both diseases.
Oncologic resection requires the removal of the involved segment of bowel,
which may require an en bloc resection of adherent structures, as well as
removal of the blood supply and lymphatics at the origin of the primary
feeding vessel.

A minimum of 12 lymph nodes should be harvested for the operation to be


considered adequate.

Adequate margins are an important component of surgery.

For colon cancer, a 5-cm margin for both proximal and distal is considered
adequate, whereas for
rectal cancer,
- adequate margin includes a 2-cm distal margin for those who did not
receive neoadjuvant chemoradiotherapy (neoChemoXRT)
- and a 1-cm distal margin for those who did receive neoChemoXRT.

Circumferential radial margin (CRM) plays a role in both colon and rectal
cancer.
For colon cancer, CRM relates to the nonperitonealized (part of the colon
that is attached to the retroperitoneum) portion of the colon, which includes
- the cecum,
- ascending colon,
- descending colon, and
- upper rectum.

For the rectum, the CRM is the nonperitonealized surface of the rectal
specimen created by the mesorectal dissection.

Although the ideal CRM is not uniformly accepted by all, in general, a CRM
greater than 1 mm can be considered adequate.

For rectal cancer, when the CRM is less than 1 mm, local recurrence rate is
22%, but when CRM is greater than 1 mm, this rate drops precipitously to
5%.
CRM less than 1 mm was also predictive of an increased risk of developing
distant disease and shorter survival.

Other studies have found that the local recurrence rate was 16% for CRM
less than 2 mm but only 6% when CRM greater than 2 mm.

Patients with colon cancer and rectal cancer are also treated differently.
For patients with stage I colon and rectal cancer, surgical resection alone is
adequate treatment.

For the majority of patients with stage II colon cancer, surgery alone is
adequate treatment.

Adjuvant chemotherapy may be considered in a select group of patients with


stage II colon cancer, including those with
(1) poorly differentiated histology who do not have high microsatellite
instability (MSI-high),
(2) bowel obstruction,
(3) bowel perforation,
(4) T4 lesions,
(5) inadequate nodal dissection (<12 lymph nodes retrieved), or
(6) evidence of peritumoral lymphatic/vascular invasion.

Adjuvant chemotherapy is used for patients with stage III colon cancer.

For patients with clinical stage II or III rectal adenocarcinoma, the treatment
of choice is neoadjuvant chemoradiotherapy followed by surgery
(neoadjuvant approach).

Compared with surgery followed by adjuvant chemoradiotherapy, the


neoadjuvant approach significantly decreases local recurrence rate, although
there were no significant differences in overall survival, disease-free
survival, or distant relapse rate between the two approaches.

The laparoscopic approach is widely considered to be oncologically


equivalent to the open approach in patients with colon cancer, but recent
data suggest this equivalency does not extend to rectal cancer.

Bevacizumab, a recombinant humanized monoclonal antibody that blocks


angiogenesis by inhibiting vascular endothelial growth factor A (VEGF-A),
does not significantly improve survival in the adjuvant setting for patients
with colorectal cancer.

==============================================================================
SURGICAL REVIEW
Once the diagnosis of HCC is established, the choice of therapy must be individualized to each
patient and based on
- tumor burden,
- presence of underlying liver disease,
- patient performance status, and
- the overall possibility of side effects or complications balanced with acceptable results.
When feasible, anatomic resection is the treatment of choice in patients without liver disease and
appears to be superior to simple wedge resection.

There is a growing body of evidence that RFA may be used in select patients with similar survival
benefit to surgical resection.
Feng et al. randomized 168 patients with small (<4 cm) hepatocellular carcinomas to surgical
resection or RFA.
There was no statistical difference in survival between the two groups, though complications
were significantly lower in the RFA group.
That being said, locoregional therapies (RFA, irreversible electroporation, proton beam therapy)
are typically reserved for tumors that are not amenable to surgical resection or as bridge therapy
to transplant (C).
The best results have been seen with tumors that are less than 4 cm in size.

Irreversible electroporation (Nanoknife) therapies show some promise but are still not included in
the current National Comprehensive Cancer Network (NCCN) guidelines for treatment of
hepatocellular carcinoma (D).

Patients with liver disease and elevated bilirubin are less likely to tolerate any surgical
intervention.
In fact, the Barcelona Clinic Liver Cancer group identified the absence of clinically relevant portal
hypertension and normal bilirubin level as major determinants for successful liver resection (B).

The only treatment modality left then to cirrhotics with HCC is liver transplantation.

The most widely used standard to choose appropriate patients is known as the Milan criteria, and
it is used by United Network for Organ Sharing (UNOS) to select candidates.
The Milan criteria are as follows:
- single tumor less than or equal to 5 cm or
- up to three tumors with none larger than 3 cm, and
- no evidence of vascular invasion, regional lymphadenopathy, or distant disease.

TACE is another useful therapy for individuals not eligible for resection or regional treatment due
to severity of their cirrhosis or other comorbidities (A).
However, it is still contraindicated in Child class C cirrhosis or for cases in which the location
precludes selective treatment.

The only chemotherapy currently approved for HCC is sorafenib, which has been shown to
slightly improve survival from 7.9 to 10.7 months.
==============================================================================

UV radiation is a known risk factor for


- squamous cell carcinoma,
- basal cell carcinoma, and
- possibly malignant melanoma.

It acts as both an initiator and promoter of direct DNA damage and damage of DNA repair
mechanisms.
The degree of risk depends on the type of UV rays and the intensity of exposure.

A higher quantity of melanin in skin is protective (C).

The UV portion of the electromagnetic spectrum can be divided into three wavelength ranges—
UVA (320–400 nm), UVB (280–320 nm), and UVC (200–280 nm).
Of these, UVB is the most significant contributor to skin damage (B).

The mechanism of carcinogenicity by UVB is by formation of pyrimidine dimers in DNA (D).


This damage can be repaired by the nucleotide excision repair pathway.
With excessive sun exposure, it is postulated that the capacity of this pathway is overwhelmed,
and some DNA that is damaged remains unrepaired.

Mutations in the ras and p53 genes occur early in skin cancers, mainly at the dipyrimidine
sequences.

The BCL-2 gene is involved in regulating cell apoptosis (E).


==============================================================================

Despite long-standing use in the treatment of cancer, the complete mechanism of radiotherapy-
induced cancer cell death has yet to be fully elucidated.

Charged particles, usually photons, are delivered to the target cells by one of three mechanisms:
- external beam,
- brachytherapy, or
- as a radioactive isotope (e.g., iodine-131 in thyroid cancer).

These charged particles interact with the outer layer of loosely bound electrons in normal atoms.
Energy is transferred from the photon, and the electron is deflected out of orbit with a lower
energy creating a “free radical.”
This effect is called the Compton effect.
The energy dissipated by these ionizing events leads to the disruption of chemical bonds, most
importantly those in DNA.

While the ionizing radiation has a direct effect on DNA in certain cells, it also indirectly affects
other cells by forming oxygen-free radicals (A).
The most important effect seems to be the creation of double-stranded DNA breaks.
While normal cells can repair this damage to some degree, tumor cells often have damaged or
inhibited DNA repair mechanisms.
As the energy of the photon beam increases, the penetration of tissue increases.

The skin is spared by the production of higher-energy electrons that travel forward and achieve
full intensity at a depth below the skin’s surface (B).

Tissue hypoxia has been shown to significantly reduce radiation damage and is one of the
patient-modifiable factors that is actively being researched to improve the effectiveness of
radiotherapy.

The relative hypoxia within large tumor cells is one of the reasons they tend to be more resistant
to radiation (C).

Along this theme, systemic anemia seems to have a deleterious effect on radiotherapy and
correction before radiation therapy is helpful.

In regard to the cell cycle, M phase has been found to be the most vulnerable stage to radiation
therapy (D).
==============================================================================

Folinic acid, also known as leucovorin, is frequently given as “rescue therapy” for methotrexate
toxicity.
Folinic acid is a 5-formyl derivative of tetrahydrofolic acid that does not require the action of
dihydrofolate reductase (DHFR) for its conversion and therefore is not affected by methotrexate’s
inhibitory action on DHFR.

While the mechanism is not fully understood, proton pump inhibitors, such as omeprazole, delay
the elimination of methotrexate and can potentially increase toxicity.
These medications should be stopped during therapy, if possible (A).

Folate is the natural form of vitamin B9, while folic acid is the equivalent synthetic form.
Both are reliant on the DHFR for metabolism and will have no effect on methotrexate toxicity (C,
D).

Cobalamin, or vitamin B12, can be effective in treating megaloblastic anemia, but this will have no
effect on the myelosuppression caused by methotrexate.
==============================================================================

Metastatic spread to the adrenal glands is common with breast and lung cancer, with the latter
being more prevalent.

While breast cancer is able to spread to the brain via Batson’s plexus, the most common location
of metastatic disease is the lungs (A).
Colon cancer spreads in a predictable pattern starting with the corresponding nodal basin and
then following the portal system to the liver.
Though it is possible for colon cancer to spread to the lungs, the liver is more common (B).

Pancreatic metastases can be seen throughout the abdominal cavity, but the liver is frequently
the first location following locally invasive disease (E).

While the most common metastatic tumor of the small bowel is from melanoma, melanoma
frequently spreads to the lungs first (D).
=================================================================================

p53 is a protein encoded by tumor suppressor gene TP53 that is located on the short arm of
chromosome 17p13.1.

It is important for cell cycle regulation, DNA replication, and apoptosis in response to DNA
damage.

The p53 protein binds to sequences of DNA in the promoter region of other genes to enhance or
regulate transcription (C).

p53 typically interacts with and enhances the effects of genes involved with inhibition of cell
growth or replication (D).

Mutations in the TP53 tumor suppressor gene result in unregulated cell growth and


predisposition to the development of malignant neoplasms (E).

Li-Fraumeni syndrome is an autosomal dominant, hereditary disorder characterized by a


germline mutation of the TP53 tumor suppressor gene (A).

However, it can also arise sporadically and is seen in more than half of all human cancers.

HPV, for example, encodes the protein E6, which binds and inactivates the p53 protein.
This, in part, contributes to the development of cervical dysplasia.
=================================================================================
Bevacizumab (Avastin), is a humanized monoclonal antibody against vascular endothelial growth
factor (VEGF).

It has been shown to significantly prolong survival when added to intravenous 5-fluorouracil-
based chemotherapy in first-line chemotherapy for metastatic colorectal cancer.

Unfortunately, bevacizumab has numerous adverse effects, with delayed wound healing being
one of the most prevalent.

The inhibitory effect on VEGF receptors limits angiogenesis, which is critical in wound healing.
Potentially the most devastating complication is spontaneous bowel perforation, but this is
relatively infrequent.

The effects of the chemotherapy regimen on wound healing last about 6 months, with no studies
showing an effect on wound healing after this time period (E).

In a patient that is already showing signs of impaired wound healing, additional surgery will likely
be unhelpful and potentially deleterious, especially in the absence of clinical signs of infection (A).

Supplemental nutrition in the absence of proven nutritional deficit has not been shown to
improve wound healing (B).

Leucovorin, or folinic acid, is given in conjunction with 5-FU to reduce side effects but has no
effect on bevacizumab (C).

Cetuximab, a monoclonal antibody against epidermal growth factor receptor (EGFR) has shown
to improve survival when used with FOLFIRI compared with bevacizumab. However, wound
healing complications were found to be no different (D).
=================================================================================

Symptoms of obstruction are the initial presenting symptom in up to 8% of colorectal cancers.

Emergency surgery has been classically considered the treatment of choice in these patients.
However, in the majority of studies, emergency colorectal surgery is burdened with higher
morbidity and mortality rates when compared with elective surgery, and many patients require
temporary colostomy, which deteriorates their quality of life and becomes permanent in 10% to
40% of cases.

The aim of a temporizing stent is to avoid emergency surgery and plan for elective surgery (which
can be laparoscopic) in order to improve surgical results, obtain an accurate tumor staging
(harvest appropriate number of lymph nodes), and detect the presence of any synchronous
lesions.

Additionally, this can allow for the medical optimization of the patient’s comorbidities.

Although stenting has multiple benefits, a recent prospective randomized study demonstrated no
advantage to stenting over emergency surgery.

However, in an 87-year-old female with a recent NSTEMI, operative risk would be prohibitive.

Despite the potential immediate benefits of temporizing stents, the possible implication on long-
term results of oncologic treatment remains to be seen.

However, obstruction must still be treated surgically if stenting is not possible (A–C, E).
=================================================================================
Gastric MALT lymphoma is a subset of slow-growing non-Hodgkin lymphoma that typically occurs
in the setting of chronic H. pylori infection.

While these tumors were originally treated with surgical resection, like most lymphomas, the
focus has moved away from surgery.

Initially, systemic therapy mimicked that of other gastric lymphomas with good response rates to
systemic chemotherapy and radiotherapy alone, as opposed to surgery (B, C).

However, as the connection between H. pylori and gastric MALT lymphoma became more
apparent, initial therapy has now moved toward attempted treatment with H. pylori eradication
and reservation of chemotherapy and radiation for patients who do not respond, have
recurrence, or have metastatic disease at the time of diagnosis (A).

Zullo et al. were even able to demonstrate treatment response in H. pylori–negative patients and
advocate for a trial of eradication in all patients with gastric MALT lymphoma regardless of H.
pylori status (E).

While the role for surgical intervention is extremely limited, it remains the treatment strategy of
choice in patients with complete gastric outlet obstruction or uncontrollable bleeding.
=================================================================================

Recent literature shows a conferred survival benefit for the resection of hepatic metastases in
colorectal cancer.

Multiple high-volume centers have demonstrated 5-year survival for patients with metastatic
colorectal cancer to the liver to be 25% to 58% with resection of the metastatic lesion.

Over the last two decades, the perioperative mortality associated with hepatic resection has
fallen significantly, with most high-volume centers reporting a 30-day perioperative mortality of
less than 2%.

The presence of any of the following risk factors had a negative, and additive, effect on survival in
patients with hepatic metastases from colorectal cancer:
(1) node-positive primary tumor,
(2) disease-free interval less than 12 months,
(3) multiple liver metastases,
(4) largest hepatic metastasis greater than 5 cm, and
(5) serum carcinoembryonic antigen (CEA) level greater than 200 ng/mL.

Those with none of these risk factors have the greatest 5-year survival at 60%.

Treatment will vary depending if it is a synchronous or metachronous lesion.


Synchronous lesions can be safely treated with combined colon and liver resection, provided the
hepatic resection is limited (<3 segments).

By combining the two surgeries, initiation of adjuvant chemotherapy is quicker.

Interestingly, for synchronous rectal cancer (that is both nonobstructing, nonbleeding) with liver
metastasis, some experts are now advocating liver resection first, followed by chemoradiation
therapy (because this therapy may downstage the rectal cancer).

For metachronous disease, the timing of surgery and chemotherapy is still controversial but
seems to lean more heavily to a surgery-first treatment strategy (B).

Nordlinger and colleagues published the results of a large randomized trial comparing surgery
alone versus perioperative chemotherapy and surgery in patients with resectable liver
metastases, which showed a higher rate of complications in the preoperative chemotherapy
group and no difference in survival.

Many have used this to infer that preoperative chemotherapy is deleterious without conferred
benefit, but the study was not powered to examine survival as a primary endpoint (D). In this
potentially curable patient, surgery first is likely to confer the largest survival benefit.

Patients with unresectable disease, or other poor prognostic indicators, should be considered for
systemic chemotherapy, followed by restaging and consideration for surgical therapy (A).

Radiation is never in the treatment algorithm for colon cancer (E).


=================================================================================

In 2012, the Food and Drug Administration (FDA) approved cetuximab, an anti-EGFR monoclonal
antibody, to be used with FOLFIRI, as the first-line treatment of k-ras mutant negative (wildtype)
metastatic colorectal cancer.

This approval was largely based on the CRYSTAL trial, as well as two other supportive studies.

A statistically significant overall survival and progression-free survival were appreciated in the
cetuximab group (23.5 months vs 19.5 months).

The recommended dose and schedule for cetuximab is 400 mg/m2 administered intravenously as
a 120-minute infusion as an initial dose, followed by 250 mg/m2 infused over 30 minutes weekly in
combination with FOLFIRI.

Other studies have demonstrated the negative effects and poor response rate cetuximab has in
patients with mutations in BRAF, NRAS, and PIK3CA (B–D). 
K-ras mutations are seen in 35% to 45% of patients with colorectal cancer, and this group of
patients will not benefit from cetuximab therapy.

The most common mutation is on chromosome 12 and 13. These have also been shown to predict
treatment failure with cetuximab (E).
=====================================================================
In December 2013, the United States Preventive Services Task Force recommended that women
who have family members with breast, ovarian, fallopian tube, or peritoneal cancer be evaluated
to see if they have a family history that is associated with an increased risk of a harmful mutation
in one of the BRCA genes.

Some risk factors that increase the likelihood of having one of these harmful genes include
- breast cancer before 50 years old,
- cancer in both breasts in the same woman,
- both breast and ovarian cancers in the same family,
- multiple breast cancers,
- known BRCA in the family,
- cases of male breast cancer, and
- Ashkenazi Jewish decent (C).

The others listed may have an increased risk developing breast cancer as per the GAIL model;
however, they have no increased risk that would necessitate genetic counseling (A, E).

For adopted patients, the recommendation for genetic testing is given only if they have had
breast cancer at an age younger than 50 years (B).
=================================================================================
Score (14/28)
Female carriers of mutations in BRCA1 or BRCA2 have an increased risk of developing
breast cancer and ovarian cancer as well as other malignancies.

Prophylactic mastectomy and oophorectomy should be discussed with


all BRCA mutation carriers, but the decision regarding surgery should be made on an
individual basis.

The risk of ovarian cancer is roughly 40% to 60% for carriers of a BRCA1 mutation,
whereas the lifetime risk is closer to 15% to 20% for those with a BRCA2 mutation.
=================================================================================

To date, there has not been demonstration of improvement in disease-free survival with
neoadjuvant chemotherapy; and, similar long-term outcomes have been documented in
randomized trials.

However, neoadjuvant chemotherapy has been proven effective for reduction of tumor
size which can allow for breast conservation therapy. (Answer B) 
Current practice guidelines advocate for a “no ink on tumor” approach to partial
mastectomy (lumpectomy) (2014 Society of Surgical Oncology-American Society for
Radiation Oncology Consensus Guidelines on Margins).

Trastuzumab is only indicated in patients with HER2 positive tumors.


Patients who are HER2 positive are recommended to receive neoadjuvant
chemotherapy regardless of tumor size.
=================================================================================

Rituximab is a chimeric monoclonal antibody directed against CD20, which is found on the
surface of mature B cells in non-Hodgkin lymphoma. 

Rituximab targets CD20, a common B-cell receptor, not interleukin-2.

The use of the drug has shown no significant difference in either the short-term or long-term
side-effect rate regarding thrombotic events.

Several studies have reported that the addition of the drug to chemotherapy results in a 95%
overall response rate, including a 55% complete response rate.
=====================================================================
This patient has metastatic colon adenocarcinoma.

The FDA has approved bevacizumab, which is a monoclonal antibody directed against
vascular endothelial growth factor that has been recently studied in a variety of solid
malignancies.

When VEGF is targeted and bound to bevacizumab, it cannot stimulate the growth of
blood vessels,
thus denying tumors blood, oxygen, and other nutrients needed for growth. 
==================================================================

The use of thyroglobulin as a biomarker is most sensitive if there is no remaining normal


thyroid tissue.
=====================================================================

Monoclonal antibodies created from mouse hybridoma technology are recognized by


the immune system as foreign and elicit the production of human antimouse antibodies,
or HAMAs.
This immunogenic response usually limits murine monoclonal antibodies to a single
dose. 

Because of species-specific antigens, murine antibodies can elicit an immune response


when administered to humans, limiting their efficacy.

To limit this interaction, chimeric antibodies that are fusion proteins of different species,
including humans, have been developed.
By design, monoclonal antibodies target a specific antigen or receptor on cancer cells.

In chimeric monoclonal antibodies, the derivation of the constant region (Fc) is human.

In general, modern immunotherapies are better tolerated than most traditional


chemotherapies.

==============================================================================

Patients with familial adenomatous polyposis (FAP) should undergo screening


sigmoidoscopy yearly beginning in their preteen years.

Surgery should be performed at the time that florid polyposis is detected.

Surgical options include


- total proctocolectomy with end ileostomy,
- total proctocolectomy with ileal pouch-anal anastomosis, and
- total abdominal colectomy with ileorectal anastomosis.

Proctocolectomy with ileal pouch-anal anastomosis avoids the possibility of subsequent


development of rectal cancer.
Colectomy with ileorectal anastomosis may be appropriate, especially for patients with
mild polyposis or attenuated FAP.
However, these patient should be counseled that they remain at elevated risk for
developing rectal cancer after this procedure and should undergo endoscopic
surveillance every 6 months. 
==============================================================================

Screening for hereditary nonpolyposis colon cancer entails colonoscopy at age 20 to 25


years (repeated every 1-3 years), plus annual transvaginal ultrasound or endometrial
aspiration (females) at age 20 to 25 years.
==================================================================

CTLA-4 and PD1 pathways elicit a coinhibitory signal that prevents the immune
response.
Several tumor cells are able to activate these pathways and evade this response.

Pembrolizumab is a monoclonal antibody directed against PD1 and is indicated for


refractory metastatic melanoma. 

Blockade of the CTLA-4 pathway is the mechanism of action of ipilimumab, not


pembrolizumab.

Direct stimulation of T-cell activity is the mechanism of action of systemic cytokines,


such as interleukin-2.
Inhibition of tumor neoangiogenesis is the mechanism of action of bevacizumab.

Anti-CD20 activity is the mechanism of action of rituximab, a monoclonal antibody


indicated for non-Hodgkin lymphoma.

==================================================================
Given this is a young patient with likely medullary thyroid cancer, care must be ensured
to diagnose multiple endocrine neoplasia syndromes, which include MEN 2A and 2B.
These syndromes also present with

- hyperparathyroidism (elevated calcium levels),


- pheochromocytomas (elevated metanephrine),
- mucosal neuromas, and
- Marfanoid habitus.

MRI of the brain is useful for diagnosis of pituitary tumors, seen in MEN 1.
Gastroduodenal neuroendocrine tumors are part of MEN 1.
==================================================================

The combination of bevacizumab, fluorouracil, and oxaliplatin/irinotecan is considered


as first-line therapy for metastatic colon cancer. 

Surgical resection of the primary tumor and metastases are indicated only under
specific and very selective criteria.

In general, a surgical approach is not indicated for treatment of metastatic colon cancer,
especially as first-line treatment.

Palliative care is reserved for terminal-stage disease after therapeutic regimens have
failed or are contraindicated.

The combination of bevacizumab and gemcitabine is seldom used as first-line therapy


for metastatic colon cancer.
Fluorouracil can be used as radiosensitizing chemotherapy in rectal cancer but is not
typically used as first-line single-agent chemotherapy for metastatic colon cancer.
==================================================================

The HER2 gene encodes a 185-kDa transmembrane glycoprotein with tyrosine kinase
activity that is a member of the EGFR family.

HER2 overexpression has been found to contribute to oncogenic transformation,


tumorigenesis, and metastatic potential.

HER2 overexpression in women with breast and ovarian cancer is a negative prognostic
factor, as several studies have found a correlation between HER2 overexpression and
shorter disease-free and overall survival.
This oncogene is overexpressed in 25% to 30% of human breast and ovarian cancers.

The HER2 oncoprotein is an important antitumor target.

The humanized antiHER2 monoclonal antibody trastuzumab (Herceptin®; Genentech,


San Francisco, CA) has demonstrated activity in clinical trials in women with metastatic
breast cancer overexpressing the gene.
==============================================================================

Indications to potentially omit adjuvant radiation following breast conservation include


- women older than 70 years of age with
- stage I disease (T1N0)
- estrogen receptor (ER)–positive tumors
- who are willing to have endocrine therapy and
- have no history of prior cancers.

In the study by Hughes and colleagues, omission of adjuvant radiation did not affect overall
survival (10-year survival of 67% and 66%, respectively).
There was an increase in the recurrence-free survival (98%) in women who had adjuvant
radiation.
However, the recurrence-free survival in women who did not have adjuvant radiation was
already relatively high (90%). 

Patients should have node-negative disease to consider omission of adjuvant radiation, a T1


tumor (< 2 cm) to consider omission of adjuvant radiation, and no prior history of breast cancer
to consider omission of adjuvant radiation.
==============================================================================
This patient has hereditary nonpolyposis colorectal cancer (HNPCC), or Lynch
syndrome.

It is an autosomal dominant hereditary syndrome that predisposes patients to a wide


spectrum of cancers including colorectal cancer (CRC) without polyposis.

The diagnostic criteria (Amsterdam criteria) include

- three or more family members who have histologically verified, HNPCC-


associated cancers (one of whom is a first-degree relative of the other two);
- two or more involved generations;
- at least one individual diagnosed with CRC before age 50; and
- no individuals have FAP.
HNPCC accounts for 5-7% of CRC and results from a mutation in one of the DNA
mismatch repair genes resulting in microsatellite instability.
In contrast, FAP is caused by mutations in the tumor suppressor gene APC and is
characterized by more than a hundred adenomatous polyps of the colon and rectum.
==================================================================

Given this is a young patient with unexplained hypertension and a known adrenal mass,
there is a possibility of several genetic syndromes, including
- von Hippel Lindau,
- MEN type 2,
- neurofibromatosis, etc.
The presence of vascular lesions in the eye would raise your suspicion regarding the
presence of von Hippel Lindau syndrome in this patient.
Pancreatic neuroendocrine tumors are associated with MEN 1 syndrome, which does
not include pheochromocytoma.
==============================================================================
Oncotype DX is one of the best validated 21-gene assays using reverse transcription
polymerase chain reaction (RT-PCR) on RNA isolated from paraffin-embedded breast
tumors.

This assay provides individualized risk estimates based on measurements of 16 cancer-


related genes and five reference genes.

It was originally developed for use in ER-positive, node-negative patients by


retrospective analysis of available tumor tissue from 447 patients treated with tamoxifen
± chemotherapy in three clinical trials, and the risk of recurrence at 10 years was
quantified (recurrence score [RS]).

Independent validation was performed on 668 stage I or II, node-negative, hormone


receptor-positive patients treated with tamoxifen from the NSABP B-14 trial.

In the validation study, the RS was superior to age, tumor size, and grade in predicting
prognosis, and when subdivided into low-risk (RS <18), intermediate-risk (RS 18 to 30),
and high-risk (RS >31) groups, the 10-year disease-free survival was 69% for the high-
risk group and 93% for the low-risk group.

Further analysis of patients from the NSABP B-20 trial demonstrated that patients in the
high-risk group experienced a significant benefit from the addition of chemotherapy to
tamoxifen, whereas those in the intermediate- and low-risk groups derived little to no
benefit from chemotherapy.
This 21-gene expression assay has been accepted as a clinical practice tool for
appropriately selected patients; however, optimal therapy for patients in the
intermediate-risk group remains unclear, and this question is being tested in a
prospective trial (TAILORx) where Oncotype DX will be used to select hormone
receptor-positive, node-negative patients with an intermediate RS for randomization to
hormonal therapy or combined hormonal therapy and chemotherapy.
Additional studies have shown that the recurrence score is also prognostic and
predictive of the benefit of chemotherapy in node-positive, ER-positive patients.
==================================================================

You might also like